User Avatar
wbrasic97291
Joined
Apr 2025
Subscription
Free
PrepTests ·
PT135.S1.Q20
User Avatar
wbrasic97291
Thursday, Jul 23 2020

I think a key takeaway from this question is to not overthink the argument. Let's just look at this fundamentally.

Newspapers report more frequently on small observational studies relative to large randomized trials. Therefore, these small observational studies must sound more dramatic.

There could be so many other possibilities for why these small observational studies are reported on more often and none of them could be that they just sound more dramatic. One of which is answer choice D where the small studies just happen more often. Another could have been that the small studies are typically more accurate so newspapers choose to include those in their papers more often instead.

The conclusion drawn in this stimulus is simply unwarranted.

PrepTests ·
PT125.S4.Q9
User Avatar
wbrasic97291
Tuesday, Jun 23 2020

C is so tricky. It seems so good because the premise in the stimulus has a conjunctive sufficient condition. However, we cannot assume that no difficulty = easy.

A is good. Even though it doesn't perfectly link up the conjunctive sufficient condition to the conclusion, it doesn't go absolutely wrong anywhere like the other ACs. The question stem never asks for the perfect AC, but the one that 'most helps"

PrepTests ·
PT134.S3.Q17
User Avatar
wbrasic97291
Tuesday, Apr 21 2020

Stimulus:

Conclusion: The brain's own immune cells cause the deterioration of cognitive abilities.

Premise 1: This deterioration can be slowed by AA drugs.

Premise 2: Patients with A cannot eliminate the protein BA from their brain. Hence, the immune cells attach these proteins, but in doing so they damage the brain's cognitive abilities.

Okay so from premise 2 it seems very clear how the immune cells can screw things up for a person's cognitive functions. The immune cells are trying to do something good (get rid of BA protein), but in doing so they damage the brain's cognitive functions. However, premise 2 simply says that the deterioration can be slowed by AA drugs. How does this support the conclusion that the immune cells damage cognitive abilities? Well it doesn't.

So we need to somehow link that premise to that conclusion.

A: So now we know why the A patients cannot get rid protein BA. This does nothing to help support our conclusion.

B: When AA reduces the brain's immune cells all of sudden the patients get better in terms of their cognitive abilities. Well this is great! Now it seems that the immune cells are really the problem.

PrepTests ·
PT134.S3.Q14
User Avatar
wbrasic97291
Tuesday, Apr 21 2020

Super hard question yet JY's explanation makes the flaw seem so obvious.

Stimulus: A's argument says blah blah blah.

This conclusion is false.

His argument rests on a bad assumption.

Well couldn't the conclusion still be true even those A's premises do not support that conclusion. Just because A's argument does not support that conclusion doesn't mean that the other possible x amount premises do not support that conclusion.

C is saying that even though this conclusion may be false some of the other assumptions that the conclusion is based on may still be true. Okay, so some of the other assumptions are solid and justified. But we still have this one "dubious" assumption which the argument rests on that is horrible. So who cares about the other assumptions. When we make an argument it doesn't matter if 99% of our assumptions are solid. It's that 1% that could screw us. This AC doesn't do anything.

PrepTests ·
PT134.S3.Q11
User Avatar
wbrasic97291
Tuesday, Apr 21 2020

Read the stimulus and immediately knew the flaw was an ad hominen. Didn't see it in the ACs so I panicked and ended up getting this question wrong.

Lesson: After reading the stimulus and being 99% confident I know the answer and then not seeing that AC. Just skip it and move on. Come back later to do careful POE.

PrepTests ·
PT134.S1.Q13
User Avatar
wbrasic97291
Monday, Apr 20 2020

This is just a very hard question any way you look at it. Only spend 1.2 minutes on it so although I got it wrong I can't be too mad about the outcome. Picked my loser and moved on.

So basically what we having going on here is that two equally sized groups of joggers are being compared.

One group stretches while the other doesn't.

Both groups suffered roughly the same amount of injuries.

Therefore, stretching doesn't do shit.

There is obviously a gap somewhere because it is a weaken question. So how can we say weaken the conclusion that stretching doesn't do anything? By coming up with some reason why stretching may actually be beneficial.

A: This is saying that the sample size rate of injuries was smaller than the population size rate of injuries. Okay so what? This doesn't give us any reason to believe that stretching could actually be beneficial.

B: How many is "many?" We have no idea. Super weak wording for a strong question type.

C: You know what? After looking at this AC again I feel like this goes with the argument more than weakens it. Our conclusion is that stretching doesn't help to prevent injuries. This AC is saying that jogging would not have an impact on most jogging injuries that are suffered. So in other words, jogging doesn't help in most cases. Seems really close to what the argument is saying and therefore definitely does not weaken it.

D: This AC is so damn tricky. This is basically saying that those who stretch before jogging are those who are more susceptible to getting injured in the first place. So, maybe in this study had the group of joggers who habitually stretched actually not stretched before jogging the rate injuries would have been way higher than those who do not habitually stretch. But now since the former group stretched they have reduced the rate injuries even though it is still the same as the other group. This indicates that stretching could actually be very beneficial.

E: We do not care about the "severity of injuries." We care about preventing them all together.

PrepTests ·
PT134.S1.Q10
User Avatar
wbrasic97291
Monday, Apr 20 2020

Premise: We have many examples of people resisting tech innovations whose lives sucked without those innovations.

Conclusion: Resistance to change is a more powerful determinant of human behavior relative to safety.

Okay so somehow they throw this idea of safety into the conclusion. Where the hell does safety come from? We know nothing about safety from our premises. Maybe these people are resisting the innovations because they do not feel safe with them.

A: This is great because it brings up the possibility that the conclusion is totally screwed up. From this AC, maybe its the fear of not having job security that is causing their behavior rather than simply wanting to resist the change.

B: This seems to go with the argument if anything. This is kind of the same thing as saying that people are resistant to change.

PrepTests ·
PT133.S3.Q26
User Avatar
wbrasic97291
Thursday, Mar 19 2020

Didn't really think this question was bad. The other ACs made it even easier.

AC B: So people like Walker even more. Then it makes this whole thing even weirder and def doesn't resolve anything.

AC C: So close to 20 percent didn't know. Great. Does explain the results.

AC D: Awesome. Doesn't explain the results.

AC E: The key to eliminating this one is to remember our task. Our task is to explain the paradoxical poll results. This AC simply tells us how Walker defended himself and never even makes an attempt to resolve the paradox. Who cares about how Walker defended himself. We care about explaining the results of the poll.

PrepTests ·
PT133.S3.Q23
User Avatar
wbrasic97291
Thursday, Mar 19 2020

AC D made this question tough for me. Really couldn't identify why it was wrong even during BR although knowing AC C was definitely it.

JYs explanation for this AC is fantastic. The conclusion is concerned with all people, not the small subset of people this AC is talking about.

Lesson from AC D: know who the conclusion is concerned with and make sure the AC talks about those people.

PrepTests ·
PT133.S3.Q22
User Avatar
wbrasic97291
Thursday, Mar 19 2020

Loving seeing myself get a 5 star question correct in 1.1 minutes. Saw AC A, knew it was correct, and didn't even read the others. Great strategy to use when you know you have sound fundamentals.

AC E is attractive. It's wrong because we have no idea if on the several occasions that Franklin saved lives and on the occasions that he exceeded what is reasonably expected of a police officer overlap in any way. The stimulus says that the acts must be the same. The act must do both. Not that you can have 20 saved lived and exceed police expectations 20 times but none of the acts do both.

PrepTests ·
PT133.S3.Q21
User Avatar
wbrasic97291
Thursday, Mar 19 2020

Kind of difficult to understand the argument. The conclusion makes this argument easy though. We go from talking about weed killers and their effectiveness to saying that the data on them is misleading. So I immediately knew that the correct AC had to have something to do with data on the weed killers.

AC A and AC C: Nothing about data.

AC D: This weakens the argument. If the data came from results on tests that were similar to how the soil is in normal conditions then that would make be doubt that the data is misleading.

AC E: Does the data solely rely on one of the two forms on weed killers' molecules? We have no idea from the info presented to us in the stim.

AC B: We know from the stim that soil conditions favor which of the two weed killers will be more concentrated in the soil. So if all the data gathered on the weed killers comes from studies which have the weed killers equally concentrated in the soil then this data doesn't accurately represent real world conditions and would lead me to conclude that the data is in fact misleading.

PrepTests ·
PT133.S3.Q18
User Avatar
wbrasic97291
Thursday, Mar 19 2020

Tough question. Thought I identified the flaw and went into hunt mode. Saw AC D which was my prephrase, picked it, and moved on without reading the others.

Stimulus:

If you avoid fat then you have a better shot at avoiding HD.

If you stop eating dairy then you are less likely to eat fat.

Therefore, If you avoid dairy then you have a better shot at staying in good health.

My reasoning for D was that I saw the argument jumping from lowing chance at heart disease to maintaining good health and the two shouldn't be equated.

However, AC D says is saying that the flaw is the evidence is not relevant to the conclusion. But is it really stretch to say that if you lower you chances at HD then your probability of maintaining good health is increased? I think that evidence is actually relevant to the conclusion. Yes there is a jump, but it is reasonable assumption that the LSAT expects you to make and it goes with our common sense.

Lesson from AC D: Don't be so robotic when answering these questions that I avoid using my common sense.

AC A: This is great. This is saying that one avoiding dairy may improve your chances of avoiding HD sure, but it could also have detrimental effects elsewhere that actually could decrease your overall level of health. Def a flaw. Eliminating dairy to reduce HD does not mean that eliminating dairy will have an overall positive impact on your health.

PrepTests ·
PT133.S3.Q17
User Avatar
wbrasic97291
Thursday, Mar 19 2020

I found this question way too difficult for how difficult it is in actuality. Super cookie cutter.

Stimulus:

Education in the traditional classroom → Social process

Develop Insights → Social process

Conclusion: Education in Traditional classroom → effective

The contrapositive is effective → Education in Traditional classroom

So we can chain this up. Develop insights → Social process → Education in traditional classroom

So basically all we need to say is that if you are effective then you must develop insights which is exactly what AC D says.

The last sentence is just there to throw you off. The key to quickly getting this question right is identify that they throw this phrase "develop insights" in there with no explanation for why it is important to education being effective.

AC A: Genuine? Wrong

AC B: Has "effective" in the necessary condition when we need "ineffective" there.

AC C: Never even mentions insights or ineffectiveness.

AC E: This is easier to read if it's in the contrapositive imo. Traditional classroom → Teacher acts from outside the classroom. Like AC C. This just never mentions the conclusion or any of the main reasoning in support of that argument.

PrepTests ·
PT128.S4.P2.Q8
User Avatar
wbrasic97291
Tuesday, May 19 2020

8.

I believe A is wrong for two reasons. Firstly, "entire body of work." Where does the author explicitly say this or attempt to imply this? He never explicitly says that is for sure. And he mentions a few of his works, but the entire body of work is never implied. This is a very difficult statement to justify so I need to be weary of picking an AC like this in the future for an inference question.

Secondly, "artistic merit." Are we ever trying to gauge Allen's artistic merit? Is the author concerned with how good of an artist Allen is? I really do not think so. That is definitely not part of the author's argument.

C is justified very well from the first half of the last paragraph.

PrepTests ·
PT128.S4.P1.Q3
User Avatar
wbrasic97291
Tuesday, May 19 2020

3.

A is wrong because we only know that NZ is a the world's largest producer of a component of carpets: strong wool. This does not mean that NZ has a strong carpet industry. We have no idea what the NZ carpet industry looks like. It could suck for all we know. All we know is that NZ is the largest producer of strong wool which is used in carpets.

PrepTests ·
PT133.S3.Q14
User Avatar
wbrasic97291
Thursday, Mar 19 2020

Picked AC A, but only spend 0.8 minutes on this question so that's a positive at least.

Stimulus:

I recommend that the abandoned shoe factory site should be used as a MES.

Council members say that the courthouse would be better.

They have no evidence for this.

Therefore, my recommendation is better.

Flaw? You can't conclude from a lack of evidence that your view is better.

AC A is so tricky because of the wording. Break down:

"Asserting that a lack of evidence against a view" Okay so this is referring to the author talking about how there is no evidence that the courthouse would be better. This seems to be descriptively accurate so far.

"is proof that the view is correct." This is where it goes wrong. The author thinks the view that the courthouse is better is wrong. The author does not think that the lack of evidence for the courthouse view is evidence that the courthouse view is correct. He is totally against the courthouse view.

AC B:

"Accepting a claim" Good so far. The author accepts the claim that the MES recommendation is better.

"Because advocates of an opposing claim have not adequately defended their view." Great. The author says there is no evidence for the claim so it is wrong. Matches up pretty well.

Lesson from this question: Read the ACs more carefully and pin down the abstract language of the AC to the stimulus itself.

PrepTests ·
PT133.S3.Q11
User Avatar
wbrasic97291
Thursday, Mar 19 2020

This is a super tricky question. AC A is so appealing because it is weakly stated which is great for a NA question while B is a conditional which is typically not so great for a NA question.

There seems to be two gaps in this argument. The conclusion jumps to talking about diagnosing and doing more harm than good. I liked AC B because it talked about both while AC A never even mentioned the whole doing more harm than good thing. When in doubt pick the AC that has more do with the argument especially the conclusion than the one that doesn't. I feel like this is a trick that does me well when I'm really stuck between two ACs.

I also feel like AC B is a sufficient assumption as well bridging the gap by saying if you rely on quackery then you are more likely to do more harm than good.

Regarding AC A, we don't need people to typically browse the web in order to diagnose themselves. What if only 1/1000 people did so? Doesn't really make a different for this argument.

PrepTests ·
PT133.S3.Q8
User Avatar
wbrasic97291
Thursday, Mar 19 2020

Tough question. Eliminated B because I didn't see how businesses and factories were relevant under timed conditions.

A is definitely wrong in hindsight. So there are other significant drains. This still does not explain why cutting back on A/C wouldn't solve the blackout.

B is good. If homeowners cutting back on A/C use doesn't matter that much because most of the A/C use in the region is done by businesses and factories than that explains what the author is saying

PrepTests ·
PT133.S1.Q16
User Avatar
wbrasic97291
Wednesday, Mar 18 2020

Had a lot of trouble with this one. Invested OVER 4 minutes. Got it right, but definitely take this as a loss. Investing 4 minutes in a question is always a loss whether you got it right or not.

Stimulus:

We have this survey.

This survey shows that address changes filed with post offices and DLB over the past ten years established that those moving out of Weston outnumbered those moving in 2:1.

Conclusion: Next years census should show that the population in Weston has decreased sine the last census ten years ago.

Where's the gap? Well this survey only accounts for those people filing these changes with post offices and DLB. Maybe most people moving into Weston never did this and in reality the population has increased. So a good answer could be something that blocks this gap.

A: This is one of the worst ACs I've seen in a while. Does absolutely nothing.

B: Just because we have a pattern over the last century does not mean this pattern will continue.

C: This adresses my prephrase really well. However, two problems with it. It may weaken instead of strengthen. I say may because it uses the word "many." We have no idea how many people "many" is. 2? 3? So I doubt it even weakens the argument actually. Maybe if it said "most" instead. Nevertheless, we need a strengthener.

D: This also does a good job at addressing my prephrase and does strengthen the argument. Most of the people moving out have children with them while those moving in did not. This suggests that the population really is decreasing.

E: Who cares about their careers.

User Avatar
wbrasic97291
Wednesday, Mar 18 2020

@ said:

That's amazing! congrats! This is hopefully what I like to accomplish as well and definitely very motivating. I'd like to ask you what your study plan was and how you progressed towards your goal of 170+? I am basically starting at the same spot as you (139) but I'm not sure what I need to do next in terms of drilling, blind review, go through LSAT study schedule and learn the modules.

My study plan has varied greatly because I am a college student with a seasonal job. However, what I will say is that the first thing you should focus on is fool proofing the games. Doing this should at least get you into the 150s.

Don't just take PT after PT after PT. I've been at this for 15 months and still have close to 30 fresh PTs left.

Lastly, each and every RC/LR question that you either got wrong or gave you trouble you need to do a reflection. You'll see this will benefit you greatly as you get into the 160s. Don't just say, "oh yeah that makes perfect sense idk how I got that one wrong." You need to reflect on why the AC you picked was attractive, why it was actually wrong, why the correct AC is correct, how you will approach a problem like this next time, and how you can better eliminate wrong ACs like this next time.

PrepTests ·
PT133.S1.Q14
User Avatar
wbrasic97291
Wednesday, Mar 18 2020

Super tough question. Only got this right by POE. Eliminated B because W never talks about skeptics. We can't assume skeptics is a subset of the general public. Chin definitely disagrees, but W never takes a stance on this.

PrepTests ·
PT133.S1.Q7
User Avatar
wbrasic97291
Wednesday, Mar 18 2020

Only question I got wrong in this section and even stuck with C during BR. I knew AC B was a super weak AC for a weakening question, but my reasoning just didn't see how C weakened the argument.

My reasoning for C was that it doesn't weaken the argument because so what if the host animals didn't live in Athens during the time of the epidemic. Maybe they lived there before, gave the virus to everyone, and then left before the epidemic started.

Regarding AC B, this definitely does not weaken the argument as I can see this clearly in hindsight and knew something was fishy about this AC even during BR. I just was not confident about eliminating my initial choice of AC C. The stimulus says that many (many=some) of the victims experienced hiccups. And this AC says that some of the victims did not get the hiccups. Well the stimulus accounts for this. The stimulus never says that all the victims got the hiccups. If it did, then this weakens the argument. But it doesn't.

So AC B is definitely more irrelevant than AC C, but AC C is still pretty bad because of my reasoning above imo.

So I just finished reading the Loophole in LR by Ellen Cassidy (great book for anyone no matter where you are currently in your prep) and I thought it was necessary to share this topic covered in the book with y'all. I have been studying for a little over a year now and I never noticed this concept, but now that I do I can already tell it will make a drastic improvement on my LR score.

So in the book she basically talks about strengthening, weakening, and SA (to name a few) as powerful questions that need powerful answer choices. That being said, think about how weak and also often tempting answer choices that begin with "some", "several", "many", etc. are.

I'll give an example of the point I'm trying to make. Look up question twelve, section one on prep test 45. I still think this question is very difficult, but look at the most popular wrong AC, AC D, that 4% less people picked than the correct one. "Some of the fish." In this stimulus we were never given how many fish were affected so for all we know it could be 100k fish. Some of the fish could be 1 fish. Does that weaken the argument in any way if there were in fact 100k fish in the sample size? Hell no! You could look at those first four words of the AC and with a decent level of confidence eliminate that AC saving you a lot of time trying to parse out what it means, how it is connected to the stimulus, and if it even weakens the stimulus.

So in conclusion, be very wary when you have a "powerful" question and an AC has a word that makes it weak. Most of the time it probably does very little to the argument if anything at all. I'm not saying to eliminate it based on that fact alone, but definitely be cautious of this.

Hope this helped some of y'all who never noticed this either.

User Avatar

Tuesday, Mar 17 2020

wbrasic97291

First 170+ PT!!

Over 15 months after scoring a 139 diagnostic and over 4 months after getting my first 160+ I have finally crossed the biggest bridge. Feels so good to have put so much work into something so difficult and see the benefits...even though it's taken what feels like an eternity lol.

I know posts like this helped to encourage me before I even started studying so I thought I'd share my recent feat. If anyone wants any advice DM me as I'd be happy to provide some of the insight I've gained thus far.

Happy studying y'all!

User Avatar

Wednesday, Nov 13 2019

wbrasic97291

Finally broke into the 160s...let’s go!!!

After a year of studying starting with a 139 diag and 15 PTs, I finally broke into the 160s and dropped a 163 on PT 50. Thank god haha. Finally starting to put this all together; took me long enough. Thought I’d share my recent glorious victory.

Happy studying to y’all!

PrepTests ·
PT131.S3.Q18
User Avatar
wbrasic97291
Thursday, Aug 13 2020

Damn. Could not grasp how D is correct until I saw the words "unexpected consequences."

Premise: We get these good unexpected consequences from space exploration like x, y, and z.

Conclusion: We should continue to support it.

Regarding AC A, "Although" introduces a concession, i.e., it's a throw away statement and not part of the core argument! So AC A can't be correct as it tries to establish the conclusion on the basis of things talked about in the concession statement.

Takeaway from this question: Read carefully! No reason I should have gotten this question wrong.

PrepTests ·
PT107.S2.P1.Q6
User Avatar
wbrasic97291
Tuesday, Jun 09 2020

6 is complete bullshit in my eyes. I knew that the author did NOT think that the artists had the power to predict social changes. However, where does it say that they could anticipate later artists? All it says is that they could anticipate later developments in the arts. Can we conclude from this that they could predict up and coming artists themselves or the direction art was going or possibly this is talking about something else? I have no idea so that's why I picked D. That statement in the passage is just so vague that, even in hindsight, I do not think we can conclude that the painters could anticipate later artists.

PrepTests ·
PT129.S3.Q8
User Avatar
wbrasic97291
Saturday, Aug 08 2020

A is wrong because it never really seems like the senior guild made an argument. He simply is making a request. From what I can tell, the only argument in the stimulus is the author's.

User Avatar

Thursday, Mar 07 2019

wbrasic97291

LG Troubles

So I finished the curriculum a little over a month ago and have just started fool proofing LG 1-35 using Pacifico's method. My problem is that for most of the games I have come across thus far I have a lot of trouble with them at the start. So since I don't really know how to approach it I stop and go to JY's video and I immediately understand it after he sets up the game board. So I guess my question is how do I eliminate that gap between having no idea how to approach the game to completely understanding it after watching the set-up.

Any help is greatly appreciated!

Thanks,

Will

Confirm action

Are you sure?